pk1 Creative Commons License 2017.07.06 0 0 1328

"ebből logikusan csak az következhet"

 

Nem. Illetve ürgefagyasztáskor talán igen, de itt ürgelyukba esésről van szó.

Az alábbi dolog történik (az a Kruskal-Szekeres metrika szerinti ábrázol):

 

https://i.stack.imgur.com/XUokp.gif

 

Tehát a tőlünk t2-kor induló fényjel vele egyszerre ér a szingularitáshoz.  

 

https://physics.stackexchange.com/questions/82678/does-someone-falling-into-a-black-hole-see-the-end-of-the-universe

Előzmény: újvári(54) (1326)